K
Khách

Hãy nhập câu hỏi của bạn vào đây, nếu là tài khoản VIP, bạn sẽ được ưu tiên trả lời.

6 tháng 8 2018

Xét tam giác vuông AHC và tam giác vuông AED có:

AE = AH

\(\widehat{HAC}=\widehat{EAD}\)   (Hai góc đối đỉnh)

\(\Rightarrow\Delta AHC=\Delta AED\)   (Cạnh góc vuông và góc nhọn kề)

\(\Rightarrow AC=AD\)

Xét tam giác BDC có BA là đường cao đồng thời trung tuyến nên nó là tam giác cân. Vậy thì BA cũng là tia phân giác góc B.

Gọi H' là chân đường vuông góc hạ từ A xuống BD.

Ta thấy ngay \(\Delta H'BA=\Delta HBA\)   (Cạnh huyền góc nhọn)

Vậy thì AH' = AH

Suy ra BD là tiếp tuyến của đường tròn tâm A, bán kính AH.

6 tháng 8 2018

\(i< 3u\)

5 tháng 8 2018

minh de0 can ban dang lai cau hoi cua minh dau :)

6 tháng 11 2019

Chào bạn, hãy theo dõi lời giải của mình nhé!

\(VT=\sqrt{4\left(a^2+b^2+c^2\right)+2\Sigma_{cyc}\sqrt{\left(a^2+3b^2\right)\left(b^2+3c^2\right)}}\)

\(\ge\sqrt{4\left(a+b+c\right)^2}=2\left(a+b+c\right)\) (Bunhia)

ez to prove\(\frac{\left(a+b+c\right)^2}{3}\ge a^2+b^2+c^2\)

\(\Rightarrow\frac{\left(a+b+c\right)^4}{3}\ge27\Rightarrow a+b+c\ge3\)

Thay vào và hoàn tất chứng minh.

P/s: Bài trên có ngược dấu đấy kkk

4 tháng 8 2018

Hãy tích cho tui đi

Nếu bạn tích tui

Tui không tích lại đâu

THANKS

4 tháng 8 2018

\(2=\sqrt{4}>\sqrt{3}\)

\(6=\sqrt{36}< \sqrt{41}\)

\(7=\sqrt{49}>\sqrt{47}\)

4 tháng 8 2018

\(M=\frac{1}{a+1}+\frac{1}{b+1}+\frac{1}{c+1}\ge\frac{\left(1+1+1\right)^2}{a+b+c+3}\)

\(\ge\frac{3^2}{1+3}=\frac{9}{4}\)

=>MinM=9/4 khi a=b=c=1/3

4 tháng 8 2018

sai rồi

4 tháng 8 2018

Áp dụng BĐT AM-GM với a;b;c > 0: \(abc\le\frac{\left(a+b+c\right)^3}{27}=\frac{1}{27}\)(Vì a+b+c=1)

Với a+b; b+c; c+a > 0 (Do a,b,c > 0): \(\left(a+b\right)\left(b+c\right)\left(c+a\right)\le\frac{8\left(a+b+c\right)^3}{27}=\frac{8}{27}\)

\(\Rightarrow M=abc\left(a+b\right)\left(b+c\right)\left(c+a\right)\le\frac{1}{27}.\frac{8}{27}=\frac{8}{729}\)

Vậy Max \(M=\frac{8}{729}\). Đẳng thức xảy ra <=> \(\hept{\begin{cases}a=b=c\\a+b=b+c=c+a\\a+b+c=1\end{cases}}\Leftrightarrow a=b=c=\frac{1}{3}.\)